PT12.S4.Q13 - police published a "wanted" poster for a criminal fugitive

StopLawyingStopLawying Alum Member
edited January 2016 in Logical Reasoning 821 karma
I've done hundreds of LR questions and I was never this lost with a question. Got this right because the rest of the answers weren't compatible with the argument, but I have no idea why (C) is correct. I really would appreciate a detailed explanation for this one and a breakdown of the argument.

Comments

  • inactiveinactive Alum Member
    12637 karma
    Bump!
  • Dr. YamataDr. Yamata Member Inactive ⭐
    578 karma
    This is a good one. I can't reveal the specific content, but I'll say that there was a similar question on the June test. It's sort of like a hybrid principle/parallel/whatever question. It goes to show that the heart of Logical Reasoning is the argument structure and being able to comprehend somewhat dense information as well as process it somehow. Hehe guess we'll be doing a lot of that as lawyers

    I went back over this one, and I think the best strategy here is POE. It's easier to find the wrong ones than to pre-phrase the right one. Also, we are required to accept all the facts presented by the stimulus yet at the same time make a case for considering reporting this wanted fugitive unethical.

    A) This is the opposite. This basically states that it is unethical to CONCEAL the fugitive's identity. Bad.

    B) Anyone except personnel in the office? But the stim already said they could report gunshot wounds. This isn't compatible. Bad.

    C) Okkkk.. I'm liking this. It makes the needed exception for gunshot wounds, and also offers a reason why the action may be considered unethical that is not forbidden by the confines of the stim. I'd hold onto it.

    D) Hmm. The stim doesn't suggest that physicians would need patient consent to report the gunshot wounds. This isn't compatible, either. Bad.

    E) Similar to to B.. if this were the case, then the gunshot wound reporting would be out of line. Bad.

    So.. C is the best out of the bunch, and as it turns out, it is correct. Notice that the stim says physicians are required to report gunshot wounds to specifically police, and infectious diseases to specifically health authorities. The "poster" of the request for information was police, so the fact that C doesn't mention infectious disease reporting to health authorities doesn't necessarily trigger a contradiction.

    Great question
  • StopLawyingStopLawying Alum Member
    821 karma
    What an awesome explanation, thanks!!!
Sign In or Register to comment.